LSAT and Law School Admissions Forum

Get expert LSAT preparation and law school admissions advice from PowerScore Test Preparation.

 Sherry123
  • Posts: 2
  • Joined: Jun 05, 2016
|
#26672
Hi there,

I missed this question and was hoping to get an expert's take on the question and rationale behind the correct answer choice.

Thank you!
Sherry
 Shannon Parker
PowerScore Staff
  • PowerScore Staff
  • Posts: 147
  • Joined: Jun 08, 2016
|
#26683
Hey,

This question is pretty simple, but only once you know the trick. The trick to this is recognizing that it is a parallel reasoning question, with the reasoning in the stimulus being a flawed composition argument. The reasoning of the stimulus can be broken down as follows:
"In yesterday's council election a majority of voters supported conservative candidates," =
51% --->A, "and a majority of voters supported candidates who voted in favor of the antipollution act." = 51%--->B. "Therefore , it must be that a majority of voters in yesterday's council election supported candidates who voted in favor of the antipollution act." 51%--> AB

So, 51%--->A + 51%--B=51%AB. But we should know of the bat that this flawed because if you have 49% people supporting anything other than conservative candidates it is possible within the information give in the premises, that those 49% make up 49 of the 51% who supported for candidates who voted in favor of the antipollution act, which would then only give us 2% of people voting for conservative candidates who voted in favor of the antipollution act.

The call of the question then, is to find the answer choice that mirrors (or parallels) the flawed reasoning found in the stimulus. We can immediately eliminate answers (A), and (C) because they do not involve composition arguments. Answer (D) can be diagrammed 51%--->AB =AB--->C.
Answer (E) can be diagrammed 51%--->A. 51%--->B=C.

The diagramming here is admittedly a little unorthodox, but you can see how unlike the stimulus, answer (D) involves a premise where 51% already has AB and then shifting AB to something else (being the most frequently ordered). However this answer choice is subject to the flaw of shifting populations, it goes from "customers who regularly eat" there, to everyone.

Answer choice (E) simply says that 51%--->A (cook well). It then leaves A completely alone and says that because of B, C will be true. However C does not naturally follow from B. For instance, maybe no one in Gina's house has time to cook, or maybe the one person in the house whose job it is to cook, is the bad cook. Either way this answer choice is subject to a different flaw than the stimulus.

That leaves us with answer choice (B). "According to Sara, most children like pies." 51%--->A. "According to Robert. most children like blueberries." 51%--->B. "most children like pies that contain blueberries." 51%--->AB. The reasoning is the same as the stimulus and is therefore the correct answer.

I know diagramming can be tricky, but it's very important, and pretty much the only way to get the more complicated LR questions, especially some of the flawed reasoning, and parallel reasoning questions correct.

I hope this helps.

~Shannon
 bearcats123
  • Posts: 11
  • Joined: Oct 01, 2019
|
#80472
I thought of the flawed reasoning in the question like just because there is 51%+ of one group and 51%+ of another group, doesn't mean those two groups overlap perfectly to make it 51%+. I thought that was the flaw in the argument. I found the correct answer by limiting it down by that reasoning. Just don't know if that is a proper way to go about finding the correct answer for future questions of this nature.

Thanks!
 Paul Marsh
PowerScore Staff
  • PowerScore Staff
  • Posts: 290
  • Joined: Oct 15, 2019
|
#80639
Hey bearcats - you're exactly right! That is the Flaw in this argument. Your explanation is just another way of describing what Shannon is saying above.

Nice going on this one!

Get the most out of your LSAT Prep Plus subscription.

Analyze and track your performance with our Testing and Analytics Package.